Assignment
Active
Applying the Triangle Parts Relationship Theorem
Which triangle correctly shows that the side opposite the larger angle is the larger side?
13
13
670
5
1000
5
60°
5
5
23
30°
50°
12
10
60°
60°
o
5
o

Answers

Answer 1

Answer:c

Step-by-step explanation:

C. 5, 13,12, 67,23, right angle


Related Questions

Translate the given phrase into an algebraic expression and simplify if possible: the product of −4 and 16

Answers

Answer: -4 x 16, -48

Step-by-step explanation:

word phrase: -4 x 16

symplified = -48

product means multiply

GIVING 30 POINTS

need it RN


Which equation represents a line which is perpendicular to the line
5x + 4y = -24?

Answers

5x +4y=24 I believe would be the answer
M^perpendicular = -4/5

PLEASE PLEASE HELP ME I WILL GIVE BRAINLIEST

options box one:
graph a
Graph B

Options for box two:
1. appears to decrease more
2. Is less
3. Decreases less
4. Appears to decrease less
5. Decreases more

Answers

Answer:

1) graph b

2)appears to decrease more

Step-by-step explanation:

How do you write expanded from for 0.68 with decimals

Answers

Answer:

0.6 + 0.08

Step-by-step explanation:

Helpppppp ASAP!!!!!!!

Answers

Answer: No solution.

Answer:

x=2

Step-by-step explanation:

(-82)+(-47)-(+33)do u all know this answer

Answers

Answer:

-157

Step-by-step explanation:

Answer:  -162

Hope this helps you

Step-by-step explanation:

subtract and add

what is the slope of the line that passes through (37, -9) and (36, 81)

Answers

Answer:

m= -90 is your answer

Step-by-step explanation:

For future reference, you should try using Symbolab, it works really well and I use it ALL the time!

3/4of 16/27/23/14+12/18 using bodmas rule

Answers

Your answer :------

3/4 of 16/27+23/14+12/18
=4/9+23/14+12/18
= (56+207+84)/126
= 347/126 = 2.75 approx ans.

If it helps you mark me as brainlist

Thanks

0.079times what equals 7.9

Answers

Answer:

100

Step-by-step explanation:

A robot can complete 7 tasks in 2/3 hour. Each task takes the same amount of time. How long does it take the robot to complete one task? How many tasks can the robot complete in one hour?

Answers

Answer:

a. is 2/21 hours to complete one task

b. is 10 1/2 tasks in one hour

Robot takes 2/21 hour to complete the task.

In one hour, robot complete 21/2 task.

What is ratio?

Ratio basically compares quantities, that means it show value of one quantity with respect to other quantity.

If a and b are two values, their ratio will be a:b,

Given that,

Time taken to complete 7 tasks = 2/3 hours.

To find the time taken to complete one task and tasks that completed in one hour,

Use ratio method,

7 tasks take = 2/3 hours

1 task takes = 2/(3 x 7) = 2 / 21 hours.

In 2/3 hours = 7 tasks completed,

1 hour = 21/2 tasks.

In one hour, 21/2 tasks can be completed.

To know more about Ratio on:

https://brainly.com/question/23724140

#SPJ5

A triangular flag has an area of 462 square feet. The base is 14 feet. What is the height?

Answers

Answer: 66ft

Step-by-step explanation:

A=1/2bh

462=1/2(14)(h)

462=7h

66=h

Unit 4: Lesson 9: Parallel and Perpendicular Lines Unit Test Parallel and Perpendicular Lines does anyone have the answers for this 13 question test ??

Answers

What test is this question from?

Answer:

I need these answers and the ones he put in in the comments are all wrong

Which situations can represent the expression n+ 2? Check all that apply.
Ramya's grade increased by two points
the difference between Escher's highest score and two
the number of chapters Wally read plus two more
O two fewer than the maximum number of absences Ellie is allowed
two added to Allison's age.
the sum of Mikel's height and two

Answers

Answer:

Yes: Ramya's grade increased by two points

No: the difference between Escher's highest score and two

Yes: the number of chapters Wally read plus two more

No: O two fewer than the maximum number of absences Ellie is allowed

Yes: two added to Allison's age.

Yes: the sum of Mikel's height and two

Step-by-step explanation:

If it says yes, it is because it was adding 2. If it says no, it was either subtracting 2 or any other form of math other than adding 2.

Answer:

A,C,E,F

Step-by-step explanation:

Just shortening it up! The person above me is correct, though. Don't forget to make them brainliest (if you want to of course)! Major Big brain moment... lol!

Answer answered by Jordan

Stay Safe <3

↖(^ω^)↗

The triangular arrangement of numbers shown is known as Pascal's triangle. Use inductive reasoning to find the 6 missing numbers.

Answers

Answer:

1 5 10 10 5 1

Step-by-step explanation:

The complete question has been attached as an image.

Looking at the triangle, we see a pattern. The first level we have 1, in the next level we have 1 1, the next evel we have 1 2 1, the next level we have 1 3 3 1 and so on. From here, we see that to get the numbers of the next level, we have to write 1 as the first number, then add 1 to the next number after it in the previous level to get the second number in the next level then add the second number of the previous level to the next number beside it to get the third number in the next level and so on until you get to the last number before 1 in the previous level, add that number to 1 to get the second to the last number in the next level and finally put 1 as the last number in the next level. Now, we have

1 4 6 4 1

1 5 10 10 5 1

And that is the required set of numbers.

If m<3 =54°. find each measure.

Answers

Answer/Step-by-step explanation:

Given:

m<3 = 54°

m<2 = right angle

a. m<1 + m<2 + m<3 = 180° (angles in a straight line)

m<1 + 90° + 54° = 180° (substitution)

m<1 + 144° = 180°

m<1 = 180° - 144°

m<1 = 36°

b. m<2 = 90° (right angle)

c. m<4 = m<1 (vertical angles)

m<4 = 36° (substitution)

d. m<5 = m<2 (vertical angles)

m<5 = 90°

e. m<6 = m<3 (vertical angles)

m<6 = 54°

f. m<7 + m<6 = 180° (same side interior angles)

m<7 + 54° = 180° (substitution)

m<7 = 180 - 54

m<7 = 126°

g. m<8 = m<6 (alternate interior angles are congruent)

m<8 = 54°

h. m<9 = m<7 (vertical angles)

m<9 = 126°

i. m<10 = m<8 (vertical angles)

m<10 = 54°

j. m<11 = m<4 (alternate interior angles are congruent)

m<11 = 36° (substitution)

k. m<12 + m<11 = 180° (linear pair)

m<12 + 36° = 180° (substitution)

m<12 = 180° - 36°

m<12 = 144°

l. m<13 = m<11 (vertical angles)

m<13 = 36°

m. m<14 = m<12 (vertical angles)

m<14 = 144° (substitution)

Are the statements true or false?
Select True or False for each statement.

Statement True False

1/ 4 · 3/ 4 = 3 /4 · 1/ 4

3/4÷2/5=2/5÷3/4

Answers

false false false false false

Answer:

no the first on is true and the last one is false

Step-by-step explanation:

The equations of sides of a triangle are 2x-3y+5=0, 2x+y=7 and 2x+5y=3. Find the coordinates of the vertices.

Answers

Answer:

Vertices A B y C

A ( 2 , 3 )    B ( -1 , 1 )  C ( 3, 1 )

Step-by-step explanation:

We have to solve a two-equation system, by pair of equations as follows

equation (1)   2x - 3y + 5 = 0

equation (2)  2x + y -7    = 0

Solving this system we find let´s say vertex A

0x - 4y +12 = 0      y = 3

Then x ?

2x - 3y + 5 = 0

2x - 9 + 5 = 0     ⇒ 2x = 4     x = 2

Then vertex A ( 2 , 3 )

From equation (1) and (3)

2x - 3y + 5 = 0

2x + 5y - 3 = 0

Agan subtracting (1) - (2)

0x -8y + 8 = 0

y = 1    and      

2x + 5y = 3

2x + 5 = 3

2x = -2

x = -1

Vertex B ( -1 , 1 )

Finally from equation (2) and (3) we get the third vertex C

2x + y - 7 = 0

2x + 5y - 3 = 0

0x -4y -4 = 0

y = 1

2x + y = 7

2x + 1 = 7

2x = 6

x = 3

C ( 3 , 1 )

Participants in a psychology expirement

were able to memorize an average of M Words in

T minutes, where M is -0.00lt3 and T is 0.1t2 Find the

average

number of words memorized in 10min.

Answers

Answer:

11 words

Step-by-step explanation:

The question is poorly formatted.

The right relationship between M and t is given as:

M = −0.001t³ + 0.1t²

Required

Solve for M when t = 10

To do this, substitute 10 for t

So,we have:

M = −0.001t³ + 0.1t²

M = -0.001 * 10³ + 0.1 * 10²

M = -0.001 * 1000 + 0.1 *100

M = 1 + 10

M = 11

11 words in 10 minutes

Can you guys help me find the supplements for question 6

Answers

Answer:

d

Step-by-step explanation:

i looked it up

A human gene carries a certain disease from the mother to the child with a probability rate of 39%. That is, there is a 39% chance that the child becomes infected with the disease. Suppose a female carrier of the gene has four children. Assume that the infections of the four children are independent of one another. Find the probability that all four of the children get the disease from their mother. Round to the nearest thousandth.

Answers

Answer:

[tex]Probability = 0.023[/tex]

Step-by-step explanation:

Given

Represent the given probability with P(Gene)

[tex]P(Gene) = 39\%[/tex]

[tex]Children = 4[/tex]

Since all 4 children get the disease, the required probability is calculated as thus:

[tex]Probability = P(Gene)^4[/tex]

[tex]Probability = 39\%^4[/tex]

Convert % to decimal

[tex]Probability = 0.39^4[/tex]

[tex]Probability = 0.02313441[/tex]

[tex]Probability = 0.023[/tex] Approximated

Find the solution of the differential equation that satisfies the given initial condition. xy' + y = y2, y(1) = −5

Answers

Answer: [tex]y=\dfrac{5}{5-6x}[/tex]

Step-by-step explanation:

The given differential equation: [tex]xy' + y = y^2[/tex]

[tex]\Rightarrow\ xy'=y^2-y[/tex]

[tex]\Rightarrow\ \frac{1}{y^2-y}y'\:=\frac{1}{x}\\\\\Rightarrow\ \dfrac{1}{y(y-1)}\dfrac{dy}{dx}=\frac{1}{x}\\\\\Rightarrow\dfrac{y-(y-1)}{y(y-1)}dy=\dfrac{1}{x}dx\\\\\Rightarrow\dfrac{1}{(y-1)}dy+\dfrac{1}{y}dy=\dfrac{1}{x}dx[/tex]

Integrate both sides , we get

[tex]\int\dfrac{1}{(y-1)}dy+\int\dfrac{1}{y}dy=\dfrac{1}{x}dx\\\\\Rightarrow\ \ln(y-1)-\ln y=\ln x+c\ \ \ \ (i)[/tex]

At x=1 , y=-5 (given)

[tex]\ln(-5-1)-\ln -5=\ln 1+c\\\\\Rightarrow\ \ln (-6)-\ln(-5)=0+c\\\\\Rightarrow\ \ln(\dfrac{-6}{-5})=c\\\\\Rightarrow\ \ln(\dfrac{6}{5})=c[/tex]

[tex][\ \ln a+\ln b=\ln ab ,\ \ \ \ \ \ln a-\ln b=\ln\dfrac{a}{b}\ ][/tex]

Put value of x in (i), we get

[tex]\ln(y-1)-\ln y=\ln x+\ln (\dfrac65)\\\\\Rigtarrow\ \ln (\dfrac{y-1}{y})=\ln(\dfrac{6}{5}x)[/tex]

[tex]\Rightarrow\ 1-\dfrac{1}{y}=\dfrac{6}{5}x\Rightarrow\ \dfrac{1}{y}=1-\dfrac{6}{5}x\\\\\Rightarrow\ \dfrac{1}{y}=\dfrac{5-6x}{5}\\\\\Rightarrow\ y=\dfrac{5}{5-6x}[/tex]

hence, the required solution: [tex]y=\dfrac{5}{5-6x}[/tex]

The solution to the differential equation

[tex]xy'+y=y^2[/tex]

given the initial condition [tex]y(1)=-5[/tex]  is [tex]y=\frac{5}{5-6x}[/tex]

Given the differential equation

[tex]xy'+y=y^2[/tex]

We can rearrange it as follows:

[tex]x\frac{dy}{dx}+y=y^2\\\\x\frac{dy}{dx}=y^2-y\\\\\frac{1}{y^2-y}\frac{dy}{dx}=\frac{1}{x}\\\\\frac{1}{y^2-y}dy=\frac{1}{x}dx[/tex]

Factoring the denominators of the LHS, and decomposing into partial fractions, we get

[tex]\frac{1}{y(y-1)}dy \implies \frac{1}{(y-1)}dy+\frac{1}{y}dy[/tex]

The final rearranged equation is

[tex]\frac{1}{(y-1)}dy+\frac{1}{y}dy=\frac{1}{x}dx[/tex]

Integrating both sides;

[tex]\int\frac{1}{y-1} dy +\int\frac{1}{y}dy=\int\frac{1}{x}dx\\\\ln(y-1)-ln(y)=ln(x)+c\\\\ln(\frac{y-1}{y})=ln(x)+c[/tex]

(We made of a law of logarithms on the last line to simplify the equation)

The initial condition [tex]y(1)=-5\implies y=-5 \text{ when }x=1[/tex]

Substituting into the general solution we got earlier

[tex]ln(\frac{y-1}{y})=ln(x)+c\\\\ln(\frac{-5-1}{-5})=ln(1)+c\\\\ln(\frac{-6}{-5})=ln(1)+c \\\\(\text{since }ln(1)=0)\\\\ln(\frac{-6}{-5})=c\\\\ln(\frac{6}{5})=c[/tex]

Substituting the value of [tex]c[/tex] back into the general solution

[tex]ln(\frac{y-1}{y})=ln(x)+c\\\\ln(\frac{y-1}{y})=ln(x)+ln(\frac{6}{5})\\\\ln(\frac{y-1}{y})=ln(\frac{6x}{5})\\\\\frac{y-1}{y}=\frac{6x}{5}[/tex]

When [tex]y[/tex] is made the subject of the formula

[tex]y=\frac{5}{5-6x}[/tex]

Therefore, the solution that satisfies the initial condition [tex]y(1)=-5[/tex] is [tex]y=\frac{5}{5-6x}[/tex]

Learn more about solving differential equations here: https://brainly.com/question/4537000

Nancy has 192 golf balls.
How many dozen golf balls does she have?

Answers

Answer:

16

Step-by-step explanation:

192/12=16

hope this helps :3

if it did pls mark brainliest

Answer:16

192/12 equals 16

the height h(t) of a trianle is increasing at 2.5 cm/min, while it's area A(t) is also increasing at 4.7 cm2/min. at what rate is the base b(t) chaging when the height h=15cm and the area A= 130cm2

Answers

Answer:

The base of the triangle decreases at a rate of 2.262 centimeters per minute.

Step-by-step explanation:

From Geometry we understand that area of triangle is determined by the following expression:

[tex]A = \frac{1}{2}\cdot b\cdot h[/tex] (Eq. 1)

Where:

[tex]A[/tex] - Area of the triangle, measured in square centimeters.

[tex]b[/tex] - Base of the triangle, measured in centimeters.

[tex]h[/tex] - Height of the triangle, measured in centimeters.

By Differential Calculus we deduce an expression for the rate of change of the area in time:

[tex]\frac{dA}{dt} = \frac{1}{2}\cdot \frac{db}{dt}\cdot h + \frac{1}{2}\cdot b \cdot \frac{dh}{dt}[/tex] (Eq. 2)

Where:

[tex]\frac{dA}{dt}[/tex] - Rate of change of area in time, measured in square centimeters per minute.

[tex]\frac{db}{dt}[/tex] - Rate of change of base in time, measured in centimeters per minute.

[tex]\frac{dh}{dt}[/tex] - Rate of change of height in time, measured in centimeters per minute.

Now we clear the rate of change of base in time within (Eq, 2):

[tex]\frac{1}{2}\cdot\frac{db}{dt}\cdot h = \frac{dA}{dt}-\frac{1}{2}\cdot b\cdot \frac{dh}{dt}[/tex]

[tex]\frac{db}{dt} = \frac{2}{h}\cdot \frac{dA}{dt} -\frac{b}{h}\cdot \frac{dh}{dt}[/tex] (Eq. 3)

The base of the triangle can be found clearing respective variable within (Eq. 1):

[tex]b = \frac{2\cdot A}{h}[/tex]

If we know that [tex]A = 130\,cm^{2}[/tex], [tex]h = 15\,cm[/tex], [tex]\frac{dh}{dt} = 2.5\,\frac{cm}{min}[/tex] and [tex]\frac{dA}{dt} = 4.7\,\frac{cm^{2}}{min}[/tex], the rate of change of the base of the triangle in time is:

[tex]b = \frac{2\cdot (130\,cm^{2})}{15\,cm}[/tex]

[tex]b = 17.333\,cm[/tex]

[tex]\frac{db}{dt} = \left(\frac{2}{15\,cm}\right)\cdot \left(4.7\,\frac{cm^{2}}{min} \right) -\left(\frac{17.333\,cm}{15\,cm} \right)\cdot \left(2.5\,\frac{cm}{min} \right)[/tex]

[tex]\frac{db}{dt} = -2.262\,\frac{cm}{min}[/tex]

The base of the triangle decreases at a rate of 2.262 centimeters per minute.

PLEASE help. Will give brainliest.

Answers

Answer:

y=5

x=10 and

z=2

Step-by-step explanation:

since they are equivalance then,

for

triangle ABC and triangle DFE

AB=DF,BC=FE and AC= DE

So, AB=DF

8y-20= 4y

or, y=5

Then, BC= FE

2x+5=x+15

or, x=10

and

AC=DE

3z+9=10z-5

or, z= 2

hope u got ut.

The triangles are parallel thus their sides are equal to each other peer to peer.

So ;

[tex]x + 15 = 2x + 5[/tex]

Subtract sides -5

[tex]x + 15 - 5 = 2x + 5 - 5[/tex]

[tex]x + 10 = 2x[/tex]

Subtract sides -x

[tex]x - x + 10 = 2x - x[/tex]

[tex]x = 10[/tex]

_________________________________

[tex]4y = 8y - 20[/tex]

Subtract sides -8y

[tex]4y - 8y = 8y - 8y - 20[/tex]

[tex] - 4y = - 20[/tex]

Negatives simplifies

[tex]4y = 20[/tex]

Divided sides by 4

[tex] \frac{4}{4}y = \frac{20}{4} \\ [/tex]

[tex]y = 5[/tex]

_________________________________

[tex]10z - 5 = 3z + 9[/tex]

Plus sides 5

[tex]10z - 5 + 5 = 3z + 9 + 5[/tex]

[tex]10z = 3z + 14[/tex]

Subtract sides -3z

[tex]10z - 3z = 3z - 3z + 14[/tex]

[tex]7z = 14[/tex]

Divided sides by 7

[tex] \frac{7}{7}z = \frac{14}{7} \\ [/tex]

[tex]z = 2[/tex]

_________________________________

And we're done....♥️♥️♥️♥️♥️

Select the correct answer.

Each side of a square is

(st

5) units. Which expression can be used to represent the area of the square?

22 - 50 - 10

I2 – 50 + 10

2 – 100 - 25

22 - 10x + 25

Answers

Answer:

D.) x²-10x+25 sq. units

Step-by-step explanation:

The question is not properly written. The question should have been:

If each side of a square is (x-5) units, which expression can be used to represent the area of the square.

Area of a square = L² where:

L is the length of the square

Given

L = x-5

Required

Area of the square

Substitute the given function into the formula to get the required as shown:

Area of the square = (x-5)²

Expand

A(x) = (x-5)(x-5)

A(x) = x(x)-5x-5x-5(-5)

A(x) = x²-10x-25

Hence the area of the square is (x²-10x+25)sq. units. Option D is correct.

K^2+k=0 what does k=

Answers

Answer:

k = K^2

Step-by-step explanation:

Subtract  K^2 from both sides of the equation.

What is the answer for 8

Answers

Answer:

no they share the same y value for two different x values

According to a 2018 survey by Bankrate, 20% of adults in the United States save nothing for retirement (CNBC website). Suppose that twenty-one adults in the United States are selected randomly. What is the probability that more than five of the selected adults save nothing for retirement

Answers

Answer:

0.769296

Step-by-step explanation:

The desired probability can be calculated from binomial probability distribution because there are 21 independent trials and probability of success(saving nothing for retirement) 0.2 remains same for each trial. Here, n=21 and p=0.20. We want to compute P(X>5).

The pdf of binomial distribution is

P(X=x)=nCx(p)^x(1-p)^(n-x)

P(X>5)=1-P(X≤5)

P(X≤5)=P(X=0)+P(X=1)+P(X=2)+P(X=3)+P(X=4)+P(X=5)

P(X=0)=21C0*(0.2)^0*(1-0.2)^(21-0)=0.009223

P(X=1)=21C1*(0.2)^1*(1-0.2)^(21-1)=0.048423

P(X=2)=21C2*(0.2)^2*(1-0.2)^(21-2)=0.121057

P(X=3)=21C3*(0.2)^3*(1-0.2)^(21-3)=0.191673

P(X=4)=21C4*(0.2)^4*(1-0.2)^(21-4)=0.215632

P(X=5)=21C5*(0.2)^5*(1-0.2)^(21-5)=0.183287

P(X≤5)=0.009223+0.048423+0.121057+0.191673+0.215632+0.183287

P(X≤5)=0.769296

P(X≤5) can also be computed by using excel function BINOM.DIST(5,21,0.2,TRUE) which results in

P(X≤5)=0.769296.

The probability that more than five of the selected adults save nothing for retirement is; P(X > 5) = 0.239928

We are told that 20% of adults in the US save nothing for retirement. Thus;

p = 20% = 0.2

21 adults are selected randomly. Thus;

n = 21

Probability that more than five of the selected adults save for retirement is gotten from formula for binomial probability distribution which is;

P(X = x) = nCx × p^(x) × (1 - p)^(n - x)

Thus;

P(X > 5) = 1 - P(X ≤ 5)

Where;

P(X ≤ 5) = P(X = 0) + P(X = 1) + P(X = 2) + P(X = 3) + P(X = 4) + P(X = 5)

P(X = 0) = 21C0 × 0.2^(0) × (1 - 0.2)^(21 - 0) =

0.009223

Using online binomial probability calculator, we can find the remaining as;

P(X = 1) = 0.048423

P(X = 2) = 0.121057

P(X = 3) = 0.191673

P(X = 4) = 0.215632

P(X = 5) = 0.183287

Thus;

P(X ≤ 5) = 0.048423 + 0.121057 + 0.191673 + 0.215632 + 0.183287

P(X ≤ 5) = 0.760072

Thus;

P(X > 5) = 1 - 0.760072

P(X > 5) = 0.239928

Read more about binomial probability distribution at; https://brainly.com/question/24239758

HELPPPP PLZZZ ASAPPP!!!Look at the illustration of four letters from the American Manual Alphabet. Decide whether the description of the letter is a good definition. If not,
choose a counterexample.
The letter I is formed by sticking up the smallest finger and folding all the other fingers into the palm of your hand with the thumb folded over them.
The letter J is a counterexample

The letter y is a counterexample

The letter A is a counterexample

This is a good definition
while keeping your hand still

Answers

Answer: I think it’s J, because the picture is also holding the smallest finger up (the pinky) and the rest of the fingers are folded inside the palm of the hand and the thumb is folded over them, I hope this helps!!

Step-by-step explanation:

The letter J is a counter example. Option a is correct.

Letters of alphabet to be determine.


What are alphabets?

Alphabets are the sets of letters from A to Z.

Here, the little finger is up and all the finger is folded and the thumb folded over the three finger implies its 'J'.

Thus, the letter J is a counter example.

Learn more about alphabets here:
https://brainly.com/question/20261759

#SPJ2


8(3x-2)-8x=9(2x-6) find x

Answers

if i’m solving it correctly x = 4

Answer:

x=19

Step-by-step explanation:

8(3x-2)-8x=9(2x-6) Distribute.

24x-16-8x=18x-54   Combine like terms.

16x-16=18x-54         Subtract 16x from both sides (getting rid of a variable first  

-16x     -16x               is easier).

-16=2x-54                 Add 54 to both sides.

+54    +54

38=2x                       Divide both sides by 2.

/2   /2

19=x

Hope this helps!! Have a great day ^^

Other Questions
4 m 1 34 m 2 38 (m 1 1)? Out of these people who r ur favs?WilburSootTommyInnitTubboFundySapnapDreamGeorgeNotFoundQuackityJschaltNikkiPonkSkeppyBBHPunzaweSAMedudeHiBombTechnoBlade HELP PLEASEEName three elements present in the ingredients of the cereal drink which are not listed in the nutrition facts When the light turns green, Mark accelerates straight down the road at 1.8 m/s2 for 8 seconds. What is his final velocity at the end of those 8 seconds? Calculate the number of moles of barium chloride in 427g of a 3.17%by mass barium chloride solution?a) 6.5x10^02molb) 7.83x10^02molc) 4.31x10^02mold) 7.81x10^02mole) 4.27x10^02mol Which is the better deal per car? 12 car washes for $100 or 5 car washes for $45 What are the signs of mental abuse? Which is the role of restriction enzymes?to isolate the selected geneto cut DNA into fragments of different lengthsto move and separate the strands of DNAto join the sticky ends of DNA fragments Find the value of x and y given that p//q. Type answers alphabetically, and separate by a comma and a space Need this done fast PLEASE Select Noun Phrase if the underlined group of words is a noun phrase. Select Not a Noun Phrase if the underlined group of words is not a noun phrase. The # of electrons in S2- -34 is how to do this question plz The mapping below shows y as a function fo x? True False A parking meter at a parking garage charges $4 per hour. Every fraction of an hour is rounded up to the next hour. What is the cost for parking a car in the parking garage for 312 hours Write a for loop that reads an integer from the user, and prints the sum of numbers from 1 to that integer (inclusive). If the user enters a number less than 1, he/she should be prompted to enter a number greater or equal to 1. Help Me pleasee i dont understand What are the points for y=-3x+4 and how do I graph it Solve for s in the proportion. 1. A = {2, 4, 6, 8, 10), B = {1,2,3,4,5,6,7,8,9,10}Find A UB and An B.(7